1971 AHSME Problems/Problem 8

Revision as of 11:16, 1 August 2024 by Thepowerful456 (talk | contribs) (see also box, made stated answer the correct answer B, not C (as derived), fixed other inaccuracies)
(diff) ← Older revision | Latest revision (diff) | Newer revision → (diff)

Problem

The solution set of $6x^2+5x<4$ is the set of all values of $x$ such that

$\textbf{(A) }\textstyle -2<x<1\qquad \textbf{(B) }-\frac{4}{3}<x<\frac{1}{2}\qquad \textbf{(C) }-\frac{1}{2}<x<\frac{4}{3}\qquad \\ \textbf{(D) }x<\textstyle\frac{1}{2}\text{ or }x>-\frac{4}{3}\qquad \textbf{(E) }x<-\frac{4}{3}\text{ or }x>\frac{1}{2}$

Solution

We are solving the inequality $6x^2 + 5x - 4  < 0.$ This can be factored as \[(2x-1)(3x+4) < 0\]

The graph of this inequality is a parabola facing upwards, so the interval between the roots satisfies the equation. This interval, $(-\frac{4}{3}, \frac{1}{2})$, is answer $\boxed{\textbf{(B)}}$.

-edited by coolmath34

See Also

1971 AHSC (ProblemsAnswer KeyResources)
Preceded by
Problem 7
Followed by
Problem 9
1 2 3 4 5 6 7 8 9 10 11 12 13 14 15 16 17 18 19 20 21 22 23 24 25 26 27 28 29 30 31 32 33 34 35
All AHSME Problems and Solutions

The problems on this page are copyrighted by the Mathematical Association of America's American Mathematics Competitions. AMC logo.png